Đến nội dung

Nguyenhuyen_AG

Nguyenhuyen_AG

Đăng ký: 09-09-2010
Offline Đăng nhập: 10-01-2019 - 16:22
****-

#609425 C/m : $(xy+yz+zx)(\frac{1}{(x-y)^2} + \fra...

Gửi bởi Nguyenhuyen_AG trong 17-01-2016 - 13:26

Chứng minh này sai chỗ nào nhỉ ?

 

Giả sử $y$ là số lớn nhất trong ba số $x,\,y,\,z.$ Do đó ta có $$\frac{1}{(x-y)^{2}}+\frac{1}{(y-z)^{2}}\geq \frac{8}{(x-z)^{2}}$$

 

Chỗ này.




#609099 $\sum \frac{a}{\sqrt{b+c-a}...

Gửi bởi Nguyenhuyen_AG trong 15-01-2016 - 15:44

Cho a,b,c là độ dài ba cạnh của một tam giác. Chứng minh rằng:

$\frac{a}{\sqrt{b+c-a}}+\frac{b}{\sqrt{a+c-b}}+\frac{c}{\sqrt{a+b-c}}\geq \sqrt{a}+\sqrt{b}+\sqrt{c}$

 

Áp dụng bất đẳng thức Cauchy-Schwarz, ta có

\[\sum \frac{a}{\sqrt{b+c-a}} \geqslant \frac{(\sqrt{a}+\sqrt{b}+\sqrt{c})^2}{\displaystyle \sum \sqrt{b+c-a}}.\]

Như vậy ta chỉ cần chứng minh \[\sqrt{b+c-a} + \sqrt{a+c-b} + \sqrt{a+b-c} \leqslant \sqrt{a}+\sqrt{b}+\sqrt{c}.\]

Bất đẳng thức này hiển nhiên đúng theo đánh giá cơ bản $\sqrt{x}+\sqrt{y} \leqslant \sqrt{2(x+y)}$ trong đó $x,\,y \geqslant 0.$




#609012 $\sum \frac{(2a+b+c)^2}{2a^2+(b+c)^2}...

Gửi bởi Nguyenhuyen_AG trong 14-01-2016 - 21:46

Cho các số dương a,b,c. Chứng minh rằng:

$\frac{(2a+b+c)^2}{2a^2+(b+c)^2}+\frac{(2b+a+c)^2}{2b^2+(a+c)^2}+\frac{(2c+a+b)^2}{2c^2+(a+b)^2}\leq 8$

 

Chú ý rằng

\[\frac{4}{3}\sum\frac{4a+b+c}{a+b+c} = 8,\]

cho nên bài toán sẽ được chứng minh nếu ta chỉ ra được

\[\frac{(2a+b+c)^2}{2a^2+(b+c)^2} \leqslant \frac{4}{3}\cdot\frac{4a+b+c}{a+b+c}.\]

Bất đẳng thức này hiển nhiên đúng vì sau khi thu gọn ta được

\[(2a-b-c)^2(5a+b+c) \geqslant 0.\]

Bài toán được chứng minh.

 

Nhận xét. Bài toán này vẫn đúng trong trường hợp $a,\,b,\,c$ là các số thực và $a+b+c \ne 0.$




#609007 C/m : $(xy+yz+zx)(\frac{1}{(x-y)^2} + \fra...

Gửi bởi Nguyenhuyen_AG trong 14-01-2016 - 21:37

Cho x,y,z là các số thực không âm đôi một khác nhau 

C/m : $(xy+yz+zx)(\frac{1}{(x-y)^2} + \frac{1}{(y-z)^2} + \frac{1}{(z-x)^2})\geq 9$

 

Bất đẳng thức sai với $a=\frac{1}{2},\,b=\frac{1}{17},\,c=1.$




#608501 $P=(\sum a^2+\sum ab)(\sum \frac{1}{(...

Gửi bởi Nguyenhuyen_AG trong 11-01-2016 - 20:08

cụ thể dc k bạn? 

 

Giả sử $c=\min\{a,\,b,\,c\}$ ta đánh giá được

\[P(a,\,b,\,c) \geqslant P(a,\,b,\,0) =(a^2+ab+b^2)\left[\frac{1}{(a-b)^2}+\frac{1}{a^2}+\frac{1}{b^2}\right]= \frac{(t+1)^2(t+3) }{t},\]

với $t = \frac{(a-b)^2}{ab} > 0.$




#608494 Topic về bất đẳng thức

Gửi bởi Nguyenhuyen_AG trong 11-01-2016 - 19:55

THTT tháng 10/2015 (đã hết hạn nhận bài giải, các bạn có thể thoải mái trao đổi cách giải của mình)

Cho tam giác $ABC$. Chứng minh rằng: 

$\frac{ab+bc+ca}{p^2 +9Rr} \ge \frac{4}{5}$

 

Ta có $R = \frac{abc}{4S},\,r = \frac{2S}{a+b+c}$ và $2p=a+b+c$ do đó bất đẳng thức trên tương đương với \[\frac{ab+bc+ca}{\dfrac{(a+b+b)^2}{4} +\dfrac{9abc}{2(a+b+c)}} \ge \frac{4}{5},\] hay là \[2[ab(a+b)+bc(b+c)+ca(c+a)] \geqslant a^3+b^3+c^3+9abc, \quad (1)\] hoặc \[2[ab(a+b)+bc(b+c)+ca(c+a)-6abc] \geqslant a^3+b^3+c^3-3abc,\] tương đương với \[4c(a-b)^2+2(a+b)(a-c)(b-c) \geqslant (a+b+c)[(a-b)^2+(a-c)(b-c)],\] \[{{\left( a-b \right)}^{2}}\left( 3c-a-b \right)+\left( a+b-c \right)\left( a-c \right)\left( b-c \right)\ge 0.\]

Hiển nhiên đúng nếu ta giả sử $c = \max\{a,\,b,\,c\}$ nhưng điều này luôn thực hiện được. Vậy ta có điều phải chứng minh.

 

Nhận xét. Bất đẳng thức (1) chính là bất đẳng thức Schur bậc 3 sau khi sử dụng phép đổi biến Ravi.




#608487 $P=(\sum a^2+\sum ab)(\sum \frac{1}{(...

Gửi bởi Nguyenhuyen_AG trong 11-01-2016 - 19:44

cho a,b,c là các số thực k âm khác nhau từng đôi 1. tìm min :

$P=(\sum a^2+\sum ab)(\sum \frac{1}{(a-b)^2})$

 

Gợi ý \[\min P = \min \left \{\frac{1}{t}(t+1)^2(t+3) \right \}_{t>0} = \frac{59+11\sqrt{33}}{8}.\]




#608296 $ \sum \frac{a}{\sqrt{a^{2}...

Gửi bởi Nguyenhuyen_AG trong 10-01-2016 - 12:48

Bất đẳng thức sau đây thú vị hơn \[\frac{a^2+b^2+c^2}{ab+bc+ca} \geqslant \frac{a}{\sqrt{a^{2}+8bc}}+\frac{b}{\sqrt{b^{2}+8ac}}+\frac{c}{\sqrt{c^{2}+8ab}}.\]




#608000 $A=8a^2(\frac{1}{b^2}+\frac{1}...

Gửi bởi Nguyenhuyen_AG trong 08-01-2016 - 19:17

Từ giả thiết ta có $a^2=b^2+c^2 \geqslant \frac{1}{2}(b+c)^2$ suy ra $\frac{a}{b+c} \geqslant \frac{1}{\sqrt{2}}.$ Mặt khác theo bất đẳng thức Cauchy-Schwarz, thì

\[A \geqslant 64\left(\frac{a}{b+c}\right)^2+\frac{b+c}{a}+2016.\]

Đặt $t = \frac{a}{b+c} \geqslant \frac{1}{\sqrt{2}},$ ta cần tìm giá trị nhỏ nhất của \[f(t) = 64t^2+\frac{1}{t}.\] Công việc còn lại khá đơn giản bằng bất đẳng thức AM-GM.




#606825 CMR $\sum \frac{2a}{b+c}\geq 3+\...

Gửi bởi Nguyenhuyen_AG trong 02-01-2016 - 22:20

Cho $a,b,c$ là những số thực dương. Chứng minh rằng:

$\sum \frac{2a}{b+c}\geq 3+\frac{(a-b)^{2}+(b-c)^{2}+(c-a)^{2}}{(a+b+c)^{2}}$

 

Tổng quát, hãy tìm hằng số $k$ tốt nhất để BĐT  $\sum \frac{2a}{b+c}\geq 3+k.\frac{(a-b)^{2}+(b-c)^{2}+(c-a)^{2}}{(a+b+c)^{2}}$ đúng

 

Hằng số tốt nhất của bài này là hằng số lớn nhất. Ta có $k_{\max} = 2$ và có thể chứng minh bằng bất đẳng thức Cauchy-Schwarz, S.O.S hoặc đưa về một biến.

 

P/s. Từ bài này ta có thể suy ra được bất đẳng thức Iran TST 1996.




#606446 $\dfrac{a^2}{b}+\dfrac{b^2}...

Gửi bởi Nguyenhuyen_AG trong 01-01-2016 - 13:19

cho em hỏi là dùng holder với bộ số đúp bồ (1,1,1) với VT hả hay là với bộ mấy ạ :))) @@

 

Đánh giá như vầy

 

\[\left ( \frac{a^5}{b}+\frac{b^5}{c}+\frac{c^5}{a} \right )^3(a^9b^3+b^9c^3+c^9a^3) \geqslant (a^6+b^6+c^6)^4.\]




#606375 $\dfrac{a^2}{b}+\dfrac{b^2}...

Gửi bởi Nguyenhuyen_AG trong 31-12-2015 - 22:00

2) Cho $a,b,c>0$ và $a^6+b^6+c^6=3$. Cmr:
$$\dfrac{a^2}{b}+\dfrac{b^2}{c}+\dfrac{c^2}{a}\ge 3$$

 

Đặt $p=a+b+c,\,q=ab+bc+ca$ ta có kết quả sau \[\frac{3p^4+p^2q-21q^2}{3pq} \geqslant 3\sqrt[6]{\frac{a^6+b^6+c^6}{3}}.\]




#606370 $\dfrac{a^2}{b}+\dfrac{b^2}...

Gửi bởi Nguyenhuyen_AG trong 31-12-2015 - 21:50

Tương tự ta cũng có

Cho $a,b,c$ thực dương.Chứng minh BĐT sau:

 

 

$$\frac{a^5}{b}+\frac{b^5}{c}+\frac{c^5}{a}\geq 3\sqrt[3]{\frac{(a^6+b^6+c^6)^2}{9}}$$ (Nguyễn Thúc Vũ Hoàng)

 

Bài này thì dùng bất đẳng thức Holder kết hợp với bất đẳng thức quen thuộc \[(a^2+b^2+c^2)^2 \geqslant 3(a^3b+b^3c+c^3a).\]




#606366 $\dfrac{a^2}{b}+\dfrac{b^2}...

Gửi bởi Nguyenhuyen_AG trong 31-12-2015 - 21:39

 

1) Cho $a,b,c>0$. Cmr:
$$2\left ( \dfrac{a^2}{b}+\dfrac{b^2}{c}+\dfrac{c^2}{a} \right )+3(a+b+c)\ge \dfrac{15(a^2+b^2+c^2)}{a+b+c}$$

 

 

Bất đẳng thức chặt hơn vẫn đúng \[\frac{a^2}{b}+\frac{b^2}{c}+\frac{c^2}{a} \geq \frac{37(a^2+b^2+c^2)-19(ab+bc+ac)}{6(a+b+c)}.\]




#606300 [Đại số] THPT tháng 11: Chứng minh $a^2+b^2+c^2 \leqslant (k^2+1)(a...

Gửi bởi Nguyenhuyen_AG trong 31-12-2015 - 14:37

Đến bước này thì mình chẳng biết xử lý ra sao nữa. Bước làm tiếp theo cảm giác không chặt chẽ. 

Kiểu bài BĐT này nhìn lạ hoắc luôn, không biết do ai ra đề nhỉ?
Tiện thể mod cho mình hỏi khi nào thì công bố đáp án của bài?

 

 

Câu này do mình đề nghị, trường hợp $k=0$ thì bài toán hiển nhiên vì nó trở thành đẳng thức. Khi $k=1$ thì điều kiện trở thành \[\frac{a}{b}+\frac{b}{c}+\frac{c}{a}=5.\]

Ta cần chứng minh $a^2+b^2+c^2 \leqslant 2(ab+b+ca).$ Đây là một bất đẳng thức khá hay của anh Võ Quốc Bá Cẩn, bài toán trên được mình tổng quát từ bài này.